Difference between revisions of "2022 AIME I Problems/Problem 4"

m (Solution)
Line 42: Line 42:
  
 
~MRENTHUSIASM
 
~MRENTHUSIASM
 +
 +
== Solution 2 ==
 +
 +
First we recognize that <math>w = cis(30^{\circ})</math> and <math>z = cis(12^{\circ})</math> because the cosine and sine sums of those angles give the values of <math>w</math> and <math>z</math>, respectively. By Demoivre's theorem, <math>cis(\theta)^n = cis(n\theta)</math>. When you multiply by <math>i</math>, we can think of that as rotating the complex number 90 degrees counterclockwise in the complex plane. Therefore, by the equation we know that <math>30r + 90</math> and <math>120s</math> land on the same angle.
 +
 +
This means that:
 +
 +
<cmath>30r + 90 \equiv 120s \pmod 360</cmath>
 +
 +
Which we can simplify to
 +
 +
<cmath>r+3 \equiv 4s \pmod 12</cmath>.
 +
 +
<math>
 +
\documentclass{article}
 +
\begin{document}
 +
Table \ref{table:1} is an example of a referenced \LaTeX{} element.
 +
 +
\begin{table}[h!]
 +
\centering
 +
\begin{tabular}{||c c c||}
 +
\hline
 +
Col1 & Col2 & Col3 \\ [0.5ex]
 +
\hline\hline
 +
1 & 6 & 87837 & 787 \\
 +
2 & 7 & 78 & 5415 \\
 +
3 & 545 & 778 & 7507 \\
 +
5 & 88 & 788 & 6344 \\ [1ex]
 +
\hline
 +
\end{tabular}
 +
\caption{Table to test captions and labels.}
 +
\label{table:1}
 +
\end{table}
 +
\end{document}
 +
 +
</math>
 +
  
 
==Video Solution (Mathematical Dexterity)==
 
==Video Solution (Mathematical Dexterity)==

Revision as of 23:04, 17 February 2022

Problem

Let $w = \dfrac{\sqrt{3} + i}{2}$ and $z = \dfrac{-1 + i\sqrt{3}}{2},$ where $i = \sqrt{-1}.$ Find the number of ordered pairs $(r,s)$ of positive integers not exceeding $100$ that satisfy the equation $i \cdot w^r = z^s.$

Solution

We rewrite $w$ and $z$ in polar form: \begin{align*} w &= e^{i\cdot\frac{\pi}{6}}, \\ z &= e^{i\cdot\frac{2\pi}{3}}. \end{align*} The equation $i \cdot w^r = z^s$ becomes \begin{align*} e^{i\cdot\frac{\pi}{2}} \cdot \left(e^{i\cdot\frac{\pi}{6}}\right)^r &= \left(e^{i\cdot\frac{2\pi}{3}}\right)^s \\ e^{i\left(\frac{\pi}{2}+\frac{\pi}{6}r\right)} &= e^{i\left(\frac{2\pi}{3}s\right)} \\ \frac{\pi}{2}+\frac{\pi}{6}r &= \frac{2\pi}{3}s+2\pi k \\ 3+r &= 4s+12k \\ 3+r &= 4(s+3k). \end{align*} for some integer $k.$

Since $4\leq 3+r\leq 103$ and $4\mid 3+r,$ we conclude that \begin{align*} 3+r &\in \{4,8,12,\ldots,100\}, \\ s+3k &\in \{1,2,3,\ldots,25\}. \end{align*} Note that the values for $s+3k$ and the values for $r$ have one-to-one correspondence.

We apply casework to the values for $s+3k:$

  1. $s+3k\equiv0\pmod{3}$
  2. There are $8$ values for $s+3k,$ so there are $8$ values for $r.$ It follows that $s\equiv0\pmod{3},$ so there are $33$ values for $s.$

    There are $8\cdot33=264$ ordered pairs $(r,s)$ in this case.

  3. $s+3k\equiv1\pmod{3}$
  4. There are $9$ values for $s+3k,$ so there are $9$ values for $r.$ It follows that $s\equiv1\pmod{3},$ so there are $34$ values for $s.$

    There are $9\cdot34=306$ ordered pairs $(r,s)$ in this case.

  5. $s+3k\equiv2\pmod{3}$
  6. There are $8$ values for $s+3k,$ so there are $8$ values for $r.$ It follows that $s\equiv2\pmod{3},$ so there are $33$ values for $s.$

    There are $8\cdot33=264$ ordered pairs $(r,s)$ in this case.

Together, the answer is $264+306+264=\boxed{834}.$

~MRENTHUSIASM

Solution 2

First we recognize that $w = cis(30^{\circ})$ and $z = cis(12^{\circ})$ because the cosine and sine sums of those angles give the values of $w$ and $z$, respectively. By Demoivre's theorem, $cis(\theta)^n = cis(n\theta)$. When you multiply by $i$, we can think of that as rotating the complex number 90 degrees counterclockwise in the complex plane. Therefore, by the equation we know that $30r + 90$ and $120s$ land on the same angle.

This means that:

\[30r + 90 \equiv 120s \pmod 360\]

Which we can simplify to

\[r+3 \equiv 4s \pmod 12\].

$\documentclass{article} \begin{document} Table \ref{table:1} is an example of a referenced \LaTeX{} element.

\begin{table}[h!] \centering \begin{tabular}{||c c c||}

\hline
Col1 & Col2 & Col3 \\ [0.5ex] 
\hline\hline
1 & 6 & 87837 & 787 \\ 
2 & 7 & 78 & 5415 \\
3 & 545 & 778 & 7507 \\
5 & 88 & 788 & 6344 \\ [1ex] 
\hline

\end{tabular} \caption{Table to test captions and labels.} \label{table:1} \end{table} \end{document}$ (Error compiling LaTeX. Unknown error_msg)


Video Solution (Mathematical Dexterity)

https://www.youtube.com/watch?v=XiEaCq5jf5s

See Also

2022 AIME I (ProblemsAnswer KeyResources)
Preceded by
Problem 3
Followed by
Problem 5
1 2 3 4 5 6 7 8 9 10 11 12 13 14 15
All AIME Problems and Solutions

The problems on this page are copyrighted by the Mathematical Association of America's American Mathematics Competitions. AMC logo.png